K
Khách

Hãy nhập câu hỏi của bạn vào đây, nếu là tài khoản VIP, bạn sẽ được ưu tiên trả lời.

3 tháng 12 2018

Vì 0 ≤ a ≤ b + 1 ≤ c + 2 nên ta có a + b+c ≤ (c+2)+ (c+2) + c
<=> 1 ≤ 3c+ 4 <=> -3 ≤ 3c <=> -1≤ c
Dấu bằng xảy ra <=> a+b+c=1 và a=b +1 =c+2 <=> a=1, b=0, c=1
=> Giá trị nhỏ nhất của c = -1

27 tháng 3 2020

bạn kia tên giống bạn đặt câu hỏi thế

25 tháng 5 2021

Áp dụng BĐT cosi:

\(a\sqrt{1-b^2}=\sqrt{a^2\left(1-b^2\right)}\le\dfrac{a^2+1-b^2}{2}\)

Tương tự cx có: \(b\sqrt{1-c^2}\le\dfrac{b^2+1-c^2}{2}\)

\(c\sqrt{1-a^2}\le\dfrac{c^2+1-a^2}{2}\)

Cộng vế với vế \(\Rightarrow VT\le\dfrac{3}{2}\)

Dấu = xảy ra <=> \(\left\{{}\begin{matrix}a^2=1-b^2\\b^2=1-c^2\\c^2=1-a^2\end{matrix}\right.\) \(\Leftrightarrow a^2+b^2+c^2=3-\left(a^2+b^2+c^2\right)\)

\(\Leftrightarrow a^2+b^2+c^2=\dfrac{3}{2}\) (đpcm)

17 tháng 10 2020

2. \(BĐT\Leftrightarrow\frac{1}{1+\frac{2}{a}}+\frac{1}{1+\frac{2}{b}}+\frac{1}{1+\frac{2}{c}}\ge1\)

Đặt\(\frac{2}{a}=x;\frac{2}{b}=y;\frac{2}{c}=z\)thì \(\hept{\begin{cases}x,y,z>0\\xyz=8\end{cases}}\)

Ta cần chứng minh \(\frac{1}{1+x}+\frac{1}{1+y}+\frac{1}{1+z}\ge1\Leftrightarrow\left(yz+y+z+1\right)+\left(zx+z+x+1\right)+\left(xy+x+y+1\right)\ge xyz+\left(xy+yz+zx\right)+\left(x+y+z\right)+1\)\(\Leftrightarrow x+y+z\ge6\)(Đúng vì \(x+y+z\ge3\sqrt[3]{xyz}=6\))

Đẳng thức xảy ra khi x = y = z = 2 hay a = b = c = 1

17 tháng 10 2020

3. Ta có: \(a+b+c\le\sqrt{3}\Rightarrow\left(a+b+c\right)^2\le3\)

Ta có đánh giá quen thuộc \(\left(a+b+c\right)^2\ge3\left(ab+bc+ca\right)\)

Từ đó suy ra \(ab+bc+ca\le1\)

\(A=\frac{\sqrt{a^2+1}}{b+c}+\frac{\sqrt{b^2+1}}{c+a}+\frac{\sqrt{c^2+1}}{a+b}\ge\frac{\sqrt{a^2+ab+bc+ca}}{b+c}+\frac{\sqrt{b^2+ab+bc+ca}}{c+a}+\frac{\sqrt{c^2+ab+bc+ca}}{a+b}\)\(=\frac{\sqrt{\left(a+b\right)\left(a+c\right)}}{b+c}+\frac{\sqrt{\left(b+a\right)\left(b+c\right)}}{c+a}+\frac{\sqrt{\left(c+a\right)\left(c+b\right)}}{a+b}\ge3\sqrt[3]{\frac{\left(a+b\right)\left(b+c\right)\left(c+a\right)}{\left(a+b\right)\left(b+c\right)\left(c+a\right)}}=3\)Đẳng thức xảy ra khi \(a=b=c=\frac{1}{\sqrt{3}}\)

12 tháng 4 2016

Câu 1: xy + x - y = 4

<=> (xy + x) - (y+ 1) = 3

<=> x(y+1) - (y + 1) = 3

<=> (y + 1) (x - 1) = 3

Theo bài ra cần tìm các số nguyên dương x, y => Xét các trường hợp y + 1 nguyên dương và x -1 nguyên dương.

Mà 3 = 1 x 3 => Chỉ có thể xảy ra các trường hợp sau:

* TH1: y + 1 = 1; x - 1 = 3 => y = 0; x = 4 (loại vì y = 0)

* TH2: y + 1 = 3; x -1 = 1 => y = 2; x = 2 (t/m)

Vậy x = y = 2.

Câu 2:

Ta có:

 (a - b)/x = (b-c)/y = (c-a)/z =(a-b + b -c + c - a) (x + y + z) = 0

Vì x; y; z nguyên dương => a-b =0; b - c = 0; c- a =0 => a = b = c

5 tháng 3 2018

 \(\frac{a-b}{x}=\frac{b-c}{y}=\frac{c-a}{z}\)

19 tháng 10 2016

Đặt \(a=\sqrt{\frac{yz}{x}},b=\sqrt{\frac{zx}{y}},c=\sqrt{\frac{xy}{z}}\) \(\Rightarrow ab+bc+ac=1\)

Suy ra bài toán trở về dạng chứng minh \(\frac{1}{a^2+1}+\frac{1}{b^2+1}+\frac{1}{c^2+1}\le\frac{9}{4}\)

\(\Leftrightarrow1-\frac{a^2}{a^2+1}+1-\frac{b^2}{b^2+1}+1-\frac{c^2}{c^2+1}\le\frac{9}{4}\)

\(\Leftrightarrow\frac{a^2}{a^2+1}+\frac{b^2}{b^2+1}+\frac{c^2}{c^2+1}\ge\frac{3}{4}\)(*)

Áp dụng bất đẳng thức AM-GM ta có : 

\(\frac{a^2}{a^2+1}+\frac{b^2}{b^2+1}+\frac{c^2}{c^2+1}\ge\frac{\left(a+b+c\right)^2}{a^2+b^2+c^2+3}\)

Đặt t = a+b+c \(\Rightarrow a^2+b^2+c^2=t^2-2\)

Ta cần chứng minh \(\frac{t^2}{t^2+1}\ge\frac{3}{4}\Leftrightarrow4t^2\ge3t^2+3\Rightarrow t^2\ge3\)(Luôn đúng vì \(\left(a+b+c\right)^2\ge3\left(ab+bc+ac\right)=3\))

Vậy ta có đpcm

4 tháng 1 2020

Để câu trả lời của bạn nhanh chóng được duyệt và hiển thị, hãy gửi câu trả lời đầy đủ và không nên:

  • Yêu cầu, gợi ý các bạn khác chọn (k) đúng cho mình
  • Chỉ ghi đáp số mà không có lời giải, hoặc nội dung không liên quan đến câu hỏi.